Simulator Test 6 Solution Set Wise Solution Index:: Q.1 Q.25 Q.26 - Q.50 Q.51 - Q.75 Q.76 - Q.100

You might also like

Download as pdf or txt
Download as pdf or txt
You are on page 1of 43

Simulator Test 6 Solution

Set Wise Solution Index:

       
Set A  Set B  Set C  Set D 

       
Q.1 ­  Q.25  Q.26 – Q.50  Q.51 – Q.75  Q.76 – Q.100 

       
Q.26 – Q.50  Q.51 – Q.75  Q.76 – Q.100  Q.1 – Q.25 

       
Q.51 – Q.75  Q.76 – Q.100  Q.1 – Q.25  Q.26 – Q.50 

       
Q.76 – Q.100  Q.1 – Q.25  Q.26 – Q.50  Q.51 – Q.75 
PTS 2019 | Simulator Test 6 (Set A) - Solutions | ForumIAS
 
Q.1)
Ans) c
Source: http://164.100.47.194/loksabha/FAQ.aspx#Procedural_Devices_for_raising_matters_o
Explanation) Option (c) – Both 1 and 2, is the correct answer.
Statement 1 is correct - The time immediately following the Question Hour and laying of papers and
before any listed business is taken up in the House has come to be popularly known as the `Zero Hour'.
The term `Zero Hour' is not formally recognised in our parliamentary procedure.
By conventions and practices some other devices have also developed without having any specific
sanction of the rule book. In this category fall the Zero Hour submissions.
Statement 2 is correct - Resolutions which the House passes in the matter of control over its
own proceedings: It has the force of law and its validity cannot be challenged in any court of law. The
House, by such a Resolution, evolves, sometimes, its own procedure to meet a situation not specifically
provided for in the Rules.
Subject) Polity

Q.2)
Ans) a
Source: Ancient India, Class XI: Chapter no 8 & 9
Explanation) Option (a) – 1 only is the correct answer.
Statement 1 is correct. The early Vedic period can be considered as an egalitarian society. Women
enjoyed respectable position. Levirate and widow remarriage were allowed. There was no child marriage.
However, in the later Vedic period, the status of women declined. Child marriages became common.
Statement 2 is incorrect. The earliest coins belong to the 5 th century BCE, that were circulated for the
first time in Eastern Uttar Pradesh and Bihar. In the Vedic period, trade was conducted through barter
system and the medium of exchange was gold and cows.
Statement 3 is incorrect. During the Vedic period, there was no standing army as maintaining an army
depends on a system of taxation which was not well established. During the time of war, tribal units were
mustered for battle.
Subject) Ancient History

Q.3)
Ans) b
Relevance (Why the question has been asked): It will address price volatility of perishable commodities
like potatoes, tomatoes and onions. It is important from UPSC perspective as it was launched during last
year's budget.
Source: http://pib.nic.in/newsite/PrintRelease.aspx?relid=187372
http://www.pib.nic.in/Pressreleaseshare.aspx?PRID=1551889
http://mofpi.nic.in/Schemes/operation-greens

ForumIAS Offline
2nd Floor, IAPL House, 19, Pusa Road, Karol Bagh, New Delhi – 110005| helpdesk@forumias.academy
 
PTS 2019 | Simulator Test 6 (Set A) - Solutions | ForumIAS
 
Explanation) Option (b) is the correct answer.

In the budget speech of Union Budget 2018-19, a new Scheme Operation Greens was announced on the
line of Operation Flood with an outlay of Rs.500 crore to promote Farmer Producers Organizations, agri-
logistics, processing facilities and professional management. Accordingly, the Ministry has formulated a
scheme for integrated development of Tomato, Onion and Potato (TOP) value chain and to ensure the
availability of TOP crops throughout the country round the year without price volatility.
Subject) Social Schemes

Q.4)
Ans) c
Source: Page 52, Mahesh Barnwal
Explanation) Option (c) - Seshanchalam, Agasthyamalai, Nilgiri, is the correct answer.
Seshachalam Biosphere Reserve, is situated in the Seshanchalam hills in the southern part of the Eastren
Ghats, covering an area of 4,755.99 sq km in the districts of Chittoor and Kadapa of Andhra Pradesh. It
was designated as a Biosphere Reserve in 2010. It has large reserves of red sandalwood.
The Nilgiri Biosphere Reserve encompasses 5,520 km² in the states of Tamil Nadu, Karnataka and
Kerala. It forms an almost complete ring around the Nilgiri Pleatue. The biosphere lies between 10°50′N
and 12°16′N latitude.

Agathyamalai Biosphere Reserve straddles the border of Pathanamthitta, Kollam and


Thiruvananthapuram districts in Kerala and Tirunelveli and Kanyakumari districts in Tamil Nadu, South
India at the southern end of the Western Ghats. The Biosphere lies Between 8° 8' to 9° 10' North Latitude.
Subject) Indian Geography

Q.5)

ForumIAS Offline
2nd Floor, IAPL House, 19, Pusa Road, Karol Bagh, New Delhi – 110005| helpdesk@forumias.academy
 
PTS 2019 | Simulator Test 6 (Set A) - Solutions | ForumIAS
 
Ans) b
Source: https://www.thehindu.com/opinion/lead/Empower-not-weaken-the-CAG/article10398867.ece
https://www.thehindu.com/news/national/make-cag-accountable-to-parliament-pac/article7634207.ece
Explanation) Option (b) – 1 and 2 only is the correct answer.
Subject) Polity

Q.6)
Ans) b
Source: https://www.seedsindia.org/about-us/
https://www.preventionweb.net/organizations/1187

Explanation) Option (b) - 2 only is the correct answer.


Statement 1 is incorrect. SEEDS, a non-profit voluntary organization created in 1994 in Delhi, is a
collective endeavour of young professionals drawn from development related fields.
Statement 2 is correct. SEEDS (Sustainable Environment and Ecological Development Society) has
one ultimate goal: protecting the lives and livelihoods of people exposed to disasters. T It is involved in
research activities in community development, disaster management, environmental planning, transport
planning, and urban and regional planning. Activities are carried out on behalf of government, semi-
government and international development agencies.
Recently, SEEDS released the Face of Disasters 2019 report.

Subject) Env - International Organisations

Q.7)
Ans) c
Source:
Explanation)
Subject) Polity

Q.8)
Ans) b
Source: NCERT 11th class
Explanation)
According to Montevideo Convention on the Rights and Duties of States
The state as a person of international law should possess the following qualifications:
Permanent population, defined territory, government and capacity to enter into relations with the other
states (It is also referred as sovereignty). Independent judiciary is not an essential condition to be
considered as state. For example, England.
Subject) Polity

Q.9)

ForumIAS Offline
2nd Floor, IAPL House, 19, Pusa Road, Karol Bagh, New Delhi – 110005| helpdesk@forumias.academy
 
PTS 2019 | Simulator Test 6 (Set A) - Solutions | ForumIAS
 
Ans) a
Relevance (Why the question has been asked): Recently in the news, it will help in checking the
menace of Manual scavenging and also the workers often do not use safety gear before entering a
sewer/ manhole and often succumbs to death this robot will eliminate the need of manual scavenging.
Source: https://www.thehindu.com/life-and-style/robotic-technology-from-vimal-govind-can-provide-
succour-to-manual-scavengers/article26150824.ece
https://www.thehindu.com/news/cities/Thiruvananthapuram/bandicoot-to-the-
rescue/article22860884.ece
https://qz.com/india/1217899/bandicoot-robot-indias-answer-to-manual-scavenging-clogged-sewers/
Explanation) Option (a) is the correct answer.
Genrobotics a firm based in Thiruvananthapuram in the southern state of Kerala, has created a robot
“Bandicoot” that cleans sewers—a degrading and dangerous task otherwise performed by thousands of
manual scavengers across the country.
Bandicoot weighs around 60 kg and is customisable. The depth and diameter of the manholes vary in
different areas and so Bandicoot have options for semi and fully automatic machines and they come in a
corrosion-resistant stainless steel and carbon fibre and are dirt, water and flash proof. It can clean a
manhole in five minutes and it collects the waste with a robotic arm and uses a water jet to clear sewer
blockages. It also has hydrogen sulphate and methane gas detectors.
Subject) Social Development

Q.10)
Ans) b
Relevance (Why the question has been asked): Recently in the news, it aims to create a paradigm
shift in the student mindset and prepare them to become young innovators and entrepreneurs.
Source: http://pib.nic.in/newsite/PrintRelease.aspx?relid=184737
https://aim.gov.in/atal-tinkering-marathon.php
http://www.thefastmail.com/page/detailnews/niti-aayogs-atal-innovation-mission-and-unicef-announce-
young-champions-awards-on-occasion-of-childrens-day/70487
Explanation) Option (b) – 2 and 3 only is the correct answer.
Statement 1 is correct. Atal Tinkering Marathon objective was to encourage students to observe
community problems and develop innovative solutions.
From over 650 innovations received, Top 30 innovations were identified from 20 different States and
Union territories from across India and these Top 30 teams were awarded with several prizes including a
three month-long ATL Student Innovator Program in partnership with industry and start-up incubators,
where students were trained on business and entrepreneurship skills.
Statement 2 is incorrect. Under AIM’s (Atal Innovation Mission) Atal Tinkering Labs (ATL), had launched
a six month long nationwide challenge called the Atal Tinkering Marathon, across six different thematic
areas, namely clean energy, water resources, waste management, healthcare, smart mobility and
agri-technology.
Statement 3 is incorrect. It has been organised by NITI Aayog.

ForumIAS Offline
2nd Floor, IAPL House, 19, Pusa Road, Karol Bagh, New Delhi – 110005| helpdesk@forumias.academy
 
PTS 2019 | Simulator Test 6 (Set A) - Solutions | ForumIAS
 
Subject) Social Schemes

Q.11)
Ans) a
Source: Indian economy by Shankarganesh Page# 170 – 171
Explanation)

Subject) Indian Economy

Q.12)
Ans) d
Relevance (Why the question has been asked): Italy born Milena Salvini was honored with Padma
Shree this year for her contribution to Kathakali and promoting it in the West.
Source: http://ccrtindia.gov.in/kathakali.php
https://en.wikipedia.org/wiki/Kathakali#Songs_and_musical_instruments
Explanation) Option (d) – 1 and 3 only is the correct answer.
Statement 1 and 3 are correct. Kathakali is a blend of dance, music and acting and dramatizes stories,
which are mostly adapted from the Indian epics. Statement 1 is correct. Kathakali draws its themes from
epics and Puranas and it presents the eternal conflict between good and evil in a grand manner.
Statement 2 is incorrect. Many musical instruments are used in Kathakali. Three major drums found
are Maddalam (barrel shaped), Centa (cylindrical drum played with curved sticks) and Itaykka (Idakka,
hourglass shaped drum.
Subject) Art and Culture

Q.13)
Ans) b
Source:
https://www.academia.edu/35151572/8th_April_1929_a_milestone_in_India_s_freedom_struggle
Explanation) Option (b) is the correct answer.
The Public Safety Bill of 1929 was a repressive legislation that allowed the British authorities to detain
suspects without any trial. It was to allow the British to arbitrarily deport anti-British activities. The main
objective was to repress the growing communist activities in India.

ForumIAS Offline
2nd Floor, IAPL House, 19, Pusa Road, Karol Bagh, New Delhi – 110005| helpdesk@forumias.academy
 
PTS 2019 | Simulator Test 6 (Set A) - Solutions | ForumIAS
 
Subject) Modern History

Q.14)
Ans) d
Source: https://www.sciencedaily.com/releases/2019/04/190417153754.htm
https://www2.lbl.gov/Science-Articles/Archive/sea-carb-bish.html

Explanation) Option (d) – 1, 2 and 3 is the correct answer.


Increasing the growth of phytoplankton has been proposed as a method of sequestering CO2. The carbon
might then be locked away when the algae die and sink to the bottom of the ocean.
Statement 1 is correct. Typically, nutrients in the form of iron or urea (providing nitrogen) are added to
the sea to encourage growth. Iron fertilization is the intentional introduction of iron to the upper ocean to
stimulate a phytoplankton bloom. This is intended to enhance biological productivity, which can benefit
the marine food chain and can increase carbon dioxide removal from the atmosphere.
Another method is direct injection, which would pump liquefied carbon dioxide a thousand meters deep,
either directly from shore stations or from tankers trailing long pipes at sea. At great depths, CO2 is
denser than sea water, and it may be possible to store it on the bottom as liquid or deposits of icy hydrates.
Statement 2 is correct. This fertilization of the ocean with urea, a nitrogen rich substance, encourages
phytoplankton growth. An Australian company, Ocean Nourishment Corporation (ONC), planned to inject
hundreds of tonnes of urea into the ocean, in order to boost the growth of CO2-absorbing phytoplankton,
as a way to combat climate change.
Statement 3 is correct. Perhaps the best-known mechanism of ocean carbon sequestration is the
biological gravitation pump (BGP), which, as the name suggests, is the sinking of biological debris
vertically down the water column into the ocean's interior. Bits of zooplankton fecal matter, pieces of
phytoplankton, dead microorganisms and such aggregate into clumps that become large and heavy
enough to sink over a span of days to weeks, becoming food for deep water and bottom-dwelling
creatures.
Subject) Pollution and Climate Change

Q.15)
Ans) c
Source: https://www.thebetterindia.com/142457/raja-ram-mohan-roy/
Explanation) Option (c) – 2 and 3 only is the correct answer.
Established on January 20, 1817, the Hindu College was founded by David Hare and Raja Ram Mohan
Roy. Annie Besant was active in the freedom struggle in the 20th century during the Home Rule
Movement.
Subject) Modern History

Q.16)
Ans) a

ForumIAS Offline
2nd Floor, IAPL House, 19, Pusa Road, Karol Bagh, New Delhi – 110005| helpdesk@forumias.academy
 
PTS 2019 | Simulator Test 6 (Set A) - Solutions | ForumIAS
 
Source: http://web.worldbank.org/archive/website01061/WEB/IMAGES/VOLUME-5.PDF
Explanation) Option (a) – Direct Democracy is the correct answer.
A Gram Sabha (or village assembly) in each village or a group of villages, comprising all registered voters,
is the fundamental grassroots body meant for voters’ representation as a measure of direct democracy.
Subject) Polity

Q.17)
Ans) d
Source: https://www.thehindu.com/news/national/the-hindu-explains-why-the-15th-finance-commission-
has-some-states-riled/article23384141.ece
Explanation)
Subject) Polity

Q.18)
Ans) b
Relevance (Why the question has been asked): The convention was a talking point in and after the
case of Abhinandan’s treatment by Pakistan in the times of peace.
Source: https://en.wikipedia.org/wiki/Geneva_Conventions#Conventions
Explanation) Option (b) is correct.
The Geneva Conventions (and their Additional Protocols) are international treaties that contain the most
important rules limiting the barbarity of war. They protect people do not take part in the fighting (civilians,
medics, aid workers) and those who can no longer fight (wounded, sick and shipwrecked troops, prisoners
of war).
Subject) IR

Q.19)
Ans) a
Source: http://ncert.nic.in/textbook/textbook.htm?kegy1=5-7 Page # 57
Explanation) Option (a) is the correct answer.
Tropical Evergreen forests are found in the western slope of the Western Ghats, hills of the north-eastern
region and the Andaman and Nicobar Islands. They are found in warm and humid areas with an annual
precipitation of over 200 cm and mean annual temperature above 22C.
Tropical evergreen forests are well stratified, with layers closer to the ground and are covered with shrubs
and creepers, with short structured trees followed by tall variety of trees.
In these forests, trees reach great heights up to 60 m or above. There is no definite time for trees to shed
their leaves, flowering and fruition. As such these forests appear green all the year round. Species found
in these forests include rosewood, mahogony, aini, ebony, etc.
Subject) Indian Geography

Q.20)

ForumIAS Offline
2nd Floor, IAPL House, 19, Pusa Road, Karol Bagh, New Delhi – 110005| helpdesk@forumias.academy
 
PTS 2019 | Simulator Test 6 (Set A) - Solutions | ForumIAS
 
Ans) c
Source: https://www.thehindubusinessline.com/news/worlds-first-domestically-funded-sdg-bond-
launched-in-india/article25787841.ece
Explanation) Option (c) -1 and 3 only is the correct answer.
Wheel stands for Women Holistic Empowerment and Enhanced Livelihood (WHEEL) impact bond
statement 1 is correct: It is the world’s first domestically funded sustainable development goals bond
(SDG bond) under the Grameen Outcome Accelerated Lending (GOAL) impact bond series.
statement 2 is incorrect: ChildFund India and Grameen Impact Investments India had launched the
Women Holistic Empowerment and Enhanced Livelihood (WHEEL) impact bond
statement 3 is correct: The WHEEL Impact Bond would work towards helping 2,000 marginalised tribal
women in Maharashtra and Madhya Pradesh to become self-reliant and empowered by training them to
become poultry farmers (micro-entrepreneurs), with a specific outcome target of an average annual net
income of ₹30,000 through this intervention.
Subject) Indian Economy

Q.21)
Ans) c
Source: http://agritech.tnau.ac.in/forestry/forest_clonal_overview.html
Explanation) Option (c) - 1 and 3 only is the correct answer.
Clonal selection and deployment is receiving attention as an intensive forest management tool for
increased wood production. Many pulps, paper and other wood-based industries are now establishing
clonal forestry programme after the promulgation of 1988 National Forest Policy.
UTILITY OF CLONAL PROPAGATION IN TREES
 Production of quality planting stock: In the intensive management of forests, it is very important
of develop fast and economical methods of raising superior planting stock. The somatic
propagation of trees has an edge over raising of stock through seeds. These techniques have
also become important in forestry owing to their potential in propagating trees, which possess
superior traits, in an effort to increase volume and quality of production.
 Propagation of problem trees: There are many tree species, which normally produce little or no
viable seeds, and some tree seeds have poor germination capacity. Such trees can easily be
propagated vegetative. Propagation by vegetative means is easy, more rapid and economical
than by seed. In some tree species, germination is very poor or slow or there may exist complex
dormancy problems or the seed may lose its viability very quickly. Moreover, seedlings of many
species grow slowly and take a long time to reach marketable size. In all these cases, use of
vegetative multiplication is a more convenient method of propagation.
 Maintenance of genetic uniformity: Most of the tree species are cross-pollinated and naturally
they are highly heterozygous. The progenies of such cross-pollinated trees are not true to type
and loose many of their unique characteristics. Hence, asexual clonal propagation helps in
maintaining the genetic characteristics of each species.

ForumIAS Offline
2nd Floor, IAPL House, 19, Pusa Road, Karol Bagh, New Delhi – 110005| helpdesk@forumias.academy
 
PTS 2019 | Simulator Test 6 (Set A) - Solutions | ForumIAS
 
 Production of disease-free plants: Certain tree species are often susceptible to some pests and
diseases and others may be partially or entirely resistant. Vegetative approaches using grafting
will help to produce resistant clones.
 Early flower induction: Vegetatively propagated plants are precocious in bearing i.e. Flower
earlier than seed propagated plants. This will help to reduce the rotation of the tree species and
also to increase the productivity. This will ultimately help to speed up the reproductive cycle for
accelerated breeding and testing.
 Clonal repository: Vegetative propagation helps to preserve the genotypes in clonal banks. This
will also help to evaluate genotypes and their interaction with the environment through clonal
testing. This method of propagation will help to conserve and multiply the valuable trees in a
centralized area such as in a laboratory or green house for intensive study and breeding.
 Maintenance of genetic gain: Vegetative propagation ultimately helps to capture maximum
genetic gains when used for regeneration in operational planting programme.
Subject) Biodiversity

Q.22)
Ans) a
Source: https://www.thehindu.com/sport/football/footballs-third-umpire-all-you-need-to-know-about-
var/article22398217.ece
Explanation) Option (a) – 1 and 2 only is the correct answer.
Statement 1 is correct. VAR was used at the FIFA 2018 held in Russia.
Statement 2 is correct. Its domain has been restricted to four game-changing categories -- awarding
goals, penalty or no penalty decisions, direct red cards, and cases of mistaken identity for red or yellow
cards.
Statement 3 is incorrect. Video Assistant Referee (VAR) is the newest addition to the refereeing team
who, along with an assistant VAR, will monitor the game for "clear and obvious errors." It will be a human
referee at the end of VAR to assist on field referees.
Subject) Technology

Q.23)
Ans) d
Relevance (Why the question has been asked): Recently Institution’s Innovation Council programme
was launched under Innovation cell of Ministry of Human Resource and Development (MHRD) with a
purpose to systematically foster a culture of Innovation in all Higher Education Institutions (HEIs) across
the country.
Source: http://pib.nic.in/newsite/PrintRelease.aspx?relid=185872
https://www.mic.gov.in/
http://pib.nic.in/newsite/PrintRelease.aspx?relid=185872
http://pib.nic.in/newsite/PrintRelease.aspx?relid=183295
Explanation) Option (d) – 1, 2, 3 and 4 is the correct answer.
ForumIAS Offline
2nd Floor, IAPL House, 19, Pusa Road, Karol Bagh, New Delhi – 110005| helpdesk@forumias.academy
 
PTS 2019 | Simulator Test 6 (Set A) - Solutions | ForumIAS
 
MHRD’s Innovation Cell (MIC) is undertaking multiple major initiatives and currently implementing
programs such as Atal Ranking of Institutions on Innovation Achievements (ARIIA), Smart India
Hackathon 2019 and formation of Institution’s Innovation Council (IIC) in more than 1000 Higher
Educational Institutions.
● Option 1 is correct. Atal Ranking of Institutions on Innovation Achievements (ARIIA). It is
an initiative of Ministry of Human Resource Development (MHRD), Govt. of India to
systematically rank all major higher educational institutions and universities in India on indicators
related to “Innovation and Entrepreneurship Development” amongst students and faculties.
● Option 2 is correct. MOVE Summit. To showcase innovation and build a platform to shape the
future of mobility, NITI Aayog is proud to host the first Move Summit 2018. In New Delhi on 7th
and 8th September 2018, stakeholders from across the sectors of mobility and transportation will
gather to co-create a public interest framework to revolutionize transport. Together, government,
industry, academia, civil society and media will set the base for a transport system which is safe;
clean, shared and connected; and affordable, accessible and inclusive.
● Option 3 is correct. Smart India Hackathon (SIH) 2019 - It has been organised by MHRD,
AICTE, Inter Institutional Inclusive Innovation centre (i4c) and Persistent systems. It is a unique
Open Innovation Model for identifying new and disruptive technology innovations to solve the
challenges faced in our country. It's a non-stop product development competition, where problem
statements are posed to technology students for innovative solutions.
● Option 4 is correct. Institution Innovation Council. Ministry of Human Resource Development
has established MHRD Innovation Cell (MIC)’ to systematically foster the culture of Innovation
amongst all Higher Education Institutions. The primary mandate of MIC is to encourage, inspire
and nurture young students by supporting them to work with new ideas and transform them into
prototypes while they are informative years.
MIC has envisioned encouraging creation of ‘Institution’s Innovation Council (IICs)’ across
selected HEIs. A network of these IICs will be established to promote innovation in the Institution
through multitudinous modes leading to an innovation promotion eco-system in the campuses.
Subject) Social Schemes

Q.24)
Ans) d
Source: Spectrum, A Brief History of Modern India, Chapter 5, pg no 154
Explanation) Option (d) – Neither 1 nor 2, is the correct answer.
Both the statements are incorrect. Shimla Deputation (1906) was led by Agha Khan (not Jinnah) to
demand separate electorate (not joint) for the Muslims.
Subject) Modern History

Q.25)
Ans) c

ForumIAS Offline
2nd Floor, IAPL House, 19, Pusa Road, Karol Bagh, New Delhi – 110005| helpdesk@forumias.academy
 
PTS 2019 | Simulator Test 6 (Set A) - Solutions | ForumIAS
 
Source:
https://support.powerschool.com/repository/schoolnet/isee/pdf/S1_Primary_and_Secondary_Successio
n_D.pdf
Explanation) Option (c) – 1 and 3 only is the correct answer.
Secondary succession refers to the sequential changes in vegetation that occur after a disturbance such
as wildfires, hurricanes, floods or human alterations like urbanization. The rate of change in secondary
succession is faster than in primary successions.
The causes of these secondary succession may be natural or human-made. Secondary succession may
occur in abandoned crop fields, cutover forests, areas damaged by wind storms or floods, and other
previously occupied regions.
Examples of where primary succession may take place include the formation of new islands, on new
volcanic rock, and on land formed from glacial retreats. In primary succession, the initial conditions are
often times very harsh, with little or no soil present.
Subject) Pollution and Climate Change

Q.26)
Ans) b
Source: https://www.indianetzone.com/42/muddiman_committee.htm
Explanation) Option (b) is the correct answer.
Muddiman Committee was appointed by the Government of India at the beginning of 1924 to make an
investigation into Government of India Act of 1919, especially the growing unrest due to the issue of
dyarchy.
Subject) Modern History

Q.27)
Ans) b
Source: https://www.indianeconomy.net/glossary/skewflation/
http://www.arthapedia.in/index.php?title=Skewflation
Explanation) Option (b) is the correct answer.
Skewflation means the skewness of inflation among different sectors of the economy — some sectors
are facing huge inflation, some none and some deflation. In India, food prices rose steadily during the
last months of 2009 and the early months of 2010, even though the prices of non-food items continued
to be relatively stable. As this somewhat unusual phenomenon stubbornly persisted, and policymakers
conferred on how to bring it to an end, the term ‘skewflation’ made an appearance in internal documents
of the Government of India.
Subject) Indian Economy

Q.28)
Ans) d

ForumIAS Offline
2nd Floor, IAPL House, 19, Pusa Road, Karol Bagh, New Delhi – 110005| helpdesk@forumias.academy
 
PTS 2019 | Simulator Test 6 (Set A) - Solutions | ForumIAS
 
Source: https://www.thehindu.com/news/international/bangladesh-china-india-myanmar-bcim-
economic-corridor-no-longer-listed-under-bri-umbrella/article26971613.ece
Explanation) Option (d) – 1 and 3 only is the correct answer.
South Asia is covered by three major undertakings under Belt and Road Initiative — the China-Myanmar
Economic Corridor (CMEC), the Nepal-China Trans-Himalayan Multi-dimensional Connectivity Network,
including Nepal-China cross-border railway, as well as the China Pakistan Economic Corridor (CPEC).
India’s decision to skip the Belt and Road Forum (BRF) may have led to the exclusion of the Bangladesh-
China- India- Myanmar (BCIM) Economic corridor from the list of projects covered by the China-led Belt
and Road Initiative (BRI) umbrella.
Subject) IR

Q.29)
Ans) a
Source: https://economictimes.indiatimes.com/industry/banking/finance/banking/prime-minister-modi-
launches-apix-technology-in-singapore/articleshow/66617593.cms
Explanation) Option (a) is the correct answer.
Prime Minister Narendra Modi had launched the APIX (Application Programming Interface Exchange), a
banking technology platform designed to reach nearly two billion people worldwide who are still without
bank accounts in singapore. Designed by software experts based in Hyderabad, Colombo and London,
APIX is a sophisticated technology developed by Boston-headquartered Virtusa, especially for smaller
banks, Tier 3 and 4, to reach out to people in the remote regions.
Subject) Indian Economy

Q.30)
Ans) c
Source: https://en.m.wikipedia.org/wiki/Teesta_River
(Books like Khullar, Mahesh Barnwal have inadequate information about this river)
Explanation) The Teesta river starts at Pahunri glacier in Sikkim, and flows south through rapids in the
Sikkim Himalaya. It forms the border between Sikkim and West Bengal. In India, it flows through the cities
of Rangpo, Kalimpong, Jalpaiguri and Mekhliganj. It joins the Brahmputra (Jamuna River) from north, on
its right side, at Fulchhari in Bangladesh.
Subject) Indian Geography

Q.31)
Ans) a
Relevance (Why the question has been asked): REE are very well basic physics, the knowledge of
which is expected out of an aspirant.
Source: https://www.livescience.com/39623-facts-about-radium.html
https://en.wikipedia.org/wiki/Rare-earth_element

Explanation) Option (a) – 1 only is the correct answer.


ForumIAS Offline
2nd Floor, IAPL House, 19, Pusa Road, Karol Bagh, New Delhi – 110005| helpdesk@forumias.academy
 
PTS 2019 | Simulator Test 6 (Set A) - Solutions | ForumIAS
 
Statement 1 is incorrect. A rare-earth element (REE) or rare-earth metal (REM), as defined by IUPAC,
is one of a set of seventeen chemical elements in the periodic table, specifically the fifteen lanthanides,
as well as scandium and yttrium. Scandium and yttrium are considered rare-earth elements because they
tend to occur in the same ore deposits as the lanthanides and exhibit similar chemical properties. Rarely,
a broader definition that includes actinides may be used, since the actinides share some mineralogical,
chemical, and physical (especially electron shell configuration) characteristics. The 17 rare-earth
elements are cerium (Ce), dysprosium (Dy), erbium (Er), europium (Eu), gadolinium (Gd), holmium (Ho),
lanthanum (La), lutetium (Lu), neodymium (Nd), praseodymium (Pr), promethium (Pm), samarium (Sm),
scandium (Sc), terbium (Tb), thulium (Tm), ytterbium (Yb), and yttrium (Y).

Statement 2 is correct. Despite their name, rare-earth elements are – with the exception of the
radioactive promethium – relatively plentiful in Earth's crust, with cerium being the 25th most abundant
element at 68 parts per million, more abundant than copper. However, because of their geochemical
properties, rare-earth elements are typically dispersed and not often found concentrated in rare-earth
minerals; as a result economically exploitable ore deposits are less common. The first rare-earth mineral
discovered (1787) was gadolinite, a mineral composed of cerium, yttrium, iron, silicon, and other
elements. This mineral was extracted from a mine in the village of Ytterbyin Sweden; four of the rare-
earth elements bear names derived from this single location.

Subject) Science

Q.32)
Ans) c
Explanation)
 M1 = Currency with public + Demand deposits with the Banking system (current account, saving
account) + Other deposits with RBI; (Most Liquid)
 M2 = M1 + Savings deposits of post office savings banks
 Broad Money (M3) = M1 + Time deposits with the banking system
 M4 = M3 + All deposits with post office savings banks
Therefore, correct order in order of increasing liquidity: M4-M3-M2-M1
Subject) Indian Economy

Q.33)
Ans) a
Relevance (Why the question has been asked): Recently in the news as the CCEA gave its approval
for the creation of a corpus of Rs. 2000 crore for Agri-Market Infrastructure Fund (AMIF) to be created
with NABARD for development and up-gradation of agricultural marketing infrastructure in Gramin
Agricultural Markets and Regulated Wholesale Markets.
Source: http://www.pib.nic.in/Pressreleaseshare.aspx?PRID=1562955
http://pib.nic.in/newsite/PrintRelease.aspx?relid=176061
http://pib.nic.in/newsite/PrintRelease.aspx?relid=176062

ForumIAS Offline
2nd Floor, IAPL House, 19, Pusa Road, Karol Bagh, New Delhi – 110005| helpdesk@forumias.academy
 
PTS 2019 | Simulator Test 6 (Set A) - Solutions | ForumIAS
 
Physical Infrastructure in these markets would be strengthened using MGNREGA
Explanation) Option (a) – 1 only is the correct answer.

Statement 1 is correct. In Budget 2018-19 government announced to develop and upgrade existing
22,000 rural haats into Gramin Agricultural Markets (GrAMs) to take care of the interests of more than
86% small and marginal farmers.
It will provide farmers facility to make direct sales to consumers and bulk purchasers, moreover
an Agri-Market Infrastructure Fund with a corpus of Rs.2000 crore will be setup for developing and
upgrading agricultural marketing infrastructure in the 22000 Grameen Agricultural Markets (GrAMs) and
585APMCs
Statement 2 is incorrect. In these GrAMs, physical infrastructure will be strengthened using MGNREGA
and other Government Schemes and would be electronically linked to e-NAM and exempted from
regulations of APMCs.
Subject) Social Schemes

Q.34)
Ans) a
Source: https://www.livemint.com/Money/tuI4wvfqdbVH9nQVYC1M5I/Lessons-for-P2P-lending-in-
India.html
Explanation) Option (a) – 1 only is the correct answer.
Peer-to-peer(P2P) lending system allows individuals to borrow and lend money without the use of an
official financial institution as an intermediary.
Statement 1 is correct: In September 2017, RBI notified that these will be registered as non-banking
financial companies (NBFCs) and came out with guidelines for P2P lending platforms.
Statement 2 is incorrect: It is regulated by RBI.
Subject) Economics

Q.35)
Ans) d
Explanation) Option (d) – 1, 2, 3 and 4 is the correct answer.
Option (D) is correct

ForumIAS Offline
2nd Floor, IAPL House, 19, Pusa Road, Karol Bagh, New Delhi – 110005| helpdesk@forumias.academy
 
PTS 2019 | Simulator Test 6 (Set A) - Solutions | ForumIAS
 
http://pib.nic.in/newsite/PrintRelease.aspx?relid=188951
Statement 1 is correct: GARBH-ini - A Mission to promote Maternal and Child Health and develop
prediction tools for pre-term berth,
Statement 2 is correct: IndCEPI - A Mission to develop affordable vaccines for endemic diseases,
Statement 3 is correct: Development of Biofortified and Protein Rich wheat - contributing to POSHAN
Abhiyan.
Statement 4 is correct: Mission on Anti-Microbial Resistance for Affordable Diagnostics and
Therapeutics and
Atal JaiAnusandhan Biotech Mission includes-
1) Undertaking Nationally Relevant Technology Innovation (UNaTI), which is expected to transform
Health, Agriculture and Energy sectors during the next 5 years.
2) GARBH-ini - A Mission to promote Maternal and Child Health and develop prediction tools for pre-term
berth,
3) IndCEPI - A Mission to develop affordable vaccines for endemic diseases,
4) Development of Biofortified and Protein Rich wheat - contributing to POSHAN Abhiyan,
5) Mission on Anti-Microbial Resistance for Affordable Diagnostics and Therapeutics and
6) Clean Energy Mission - Innovative Technology interventions for Swachh Bharat.

Subject) Env - International Organisations

Q.36)
Ans) b
Relevance (Why the question has been asked):
Source:
Explanation)
Subject) Polity

Q.37)
Ans) d
Relevance (Why the question has been asked): Static question. UPSC has time and again asked
questions on famous books and authors. Can be done by elimination. One must know that Hind swaraj
was written by Mahatma Gandhi.
Source: India’s Struggle for Independence (different chapters)
Explanation) Option (d) – None of the above is the correct answer.
The Hindu was started in 1878 by G. Subramania Iyer. S Subramanya Aiyar is known during his role in
the Home Rule movement where he returned his knighthood in protest against the arrest of Annie Besant.
Bharat Sramjeebi (Indian Labor) was a monthly journal by Sasipada Bannerjee. NM Lokhanday wrote
Dinabandhu.
Hind Swaraj was written by Mahatma Gandhi.
ForumIAS Offline
2nd Floor, IAPL House, 19, Pusa Road, Karol Bagh, New Delhi – 110005| helpdesk@forumias.academy
 
PTS 2019 | Simulator Test 6 (Set A) - Solutions | ForumIAS
 
Subject) Modern History

Q.38)
Ans) c
Source: DD BASU,
http://www.legalservicesindia.com/article/1390/Role-of-Preamble-Interpretation-with-Indian-
Constitution.html
Explanation) Option (c) – Both 1 and 2 is the correct answer.
Statement 1 is correct - Kesavanada Bharati Case has created a history. It was held in this case:
 that the Preamble to the Constitution of India is a part of Constitution
 that the Preamble is not a source of power nor a source of limitations
 the Preamble has a significant role to play in the interpretation of statues, also in the interpretation
of provisions of the Constitution.
Statement 2 is correct - Majority of Kesavananda Bharati case bench has held that Preamble is the part
of the constitution and it can be amended but, Parliament cannot amend the basic features of the
preamble.
Subject) Polity

Q.39)
Ans) b
Source: https://www.thehindu.com/business/crypto-currencies-icos-under-sebi
lens/article19956679.ece
https://www.thehindubusinessline.com/opinion/columns/all-you-wanted-to-know-about-initial-coin-
offerings/article9981791.ece
Explanation) Option (b) – 2 only is the correct answer.
Statement 1 is correct: An Initial Coin Offering (ICO) is the cryptocurrency space's rough equivalent to
an IPO in the mainstream investment world. ICOs act as fundraisers of sorts; a company looking to create
a new coin, app, or service launches an ICO. Next, interested investors buy in to the offering, either with
fiat currency or with pre-existing digital tokens like ether.
Statement 2 is incorrect: ICOs are used by start-ups to bypass the rigorous and regulated capital-raising
process required by venture capitalists or banks.
Unlike an IPO, which is governed by SEBI regulations, there is no regulatory body for ICOs in India.
Subject) Indian Economy

Q.40)
Ans) b
Source: http://pib.nic.in/newsite/PrintRelease.aspx?relid=179462
Explanation) Option (b) – 1 only is the correct answer.
With a view to integrate farmers with bulk purchasers including exporters, agro- industries etc. for better
price realization through mitigation of market and price risks to the farmers and ensuring smooth agro
ForumIAS Offline
2nd Floor, IAPL House, 19, Pusa Road, Karol Bagh, New Delhi – 110005| helpdesk@forumias.academy
 
PTS 2019 | Simulator Test 6 (Set A) - Solutions | ForumIAS
 
raw material supply to the agro industries, Union Finance Minister in the budget for 2017-18 announced
preparation of a “Model Contract Farming Act”
Salient features of Model Contract Farming Act, 2018 are:

● The Act lays special emphasis on protecting the interests of the farmers, considering them as
weaker of the two parties entering into a contract.

● In addition to contract farming, services contracts all along the value chain including pre-
production, production and post-production have been included.

● “Registering and Agreement Recording Committee” or an “Officer” for the purpose at


district/block/ taluka level for online registration of sponsor and recording of agreement provided.

● Contracted produce is to be covered under crop / livestock insurance in operation.

● Contract framing to be outside the ambit of APMC Act. (Statement 1 is correct)

● No permanent structure can be developed on farmers’ land/premises (Statement 2 is incorrect)

● No right, title of interest of the land shall vest in the sponsor. (Statement 3 is incorrect)
Subject) Indian Economy

Q.41)
Ans) b
Relevance (Why the question has been asked): Antarctica is one of the Continents of the world and
UPSC must be aware of the rules, regulations, territorial claims, scientific researches and procedures
governing the continent. The Treaty remains evergreen and is mentioned every now and then in news
articles.
Source: https://www.thehindubusinessline.com/news/science/here-is-why-researchers-working-in-
polar-regions-are-coming-together/article24311301.ece
https://www.ats.aq/e/ats.htm
Explanation) Option (b) – 2 only is the correct answer.
Statement 1 is incorrect. The Antarctic Treaty and related agreements, collectively known as the
Antarctic Treaty System (ATS), regulate international relations with respect to Antarctica.
For the purposes of the treaty system, Antarctica is defined as all land and ice shelves south of 60°S
latitude.
The treaty entered into force in 1961 and currently has 53 parties. The treaty sets aside Antarctica as a
scientific preserve, establishes freedom of scientific investigation, and bans military activity on the
continent. The treaty was the first arms control agreement established during the Cold War. Since
September 2004, the Antarctic Treaty Secretariat headquarters has been located in Buenos Aires,
Argentina.
India launched its first expedition to Antarctica in 1981 and within two years signed the Antarctic Treaty
in 1983. The first Indian research base Dakshin Gangotri was established in 1983-84 and India become
a member of SCAR in 1984. At present, India has two research bases Maitri and Bharati in Antarctica.
The South Pole expedition was launched, and Indian team reached the South Pole in 2010-11.
Statement 2 is correct. Some important provisions of the Treaty:
ForumIAS Offline
2nd Floor, IAPL House, 19, Pusa Road, Karol Bagh, New Delhi – 110005| helpdesk@forumias.academy
 
PTS 2019 | Simulator Test 6 (Set A) - Solutions | ForumIAS
 
 Antarctica shall be used for peaceful purposes only (Art. I)
 Freedom of scientific investigation in Antarctica and cooperation toward that end … shall continue
(Art. II).
 Scientific observations and results from Antarctica shall be exchanged and made freely available
(Art. III).
Among the signatories of the Treaty were seven countries - Argentina, Australia, Chile, France, New
Zealand, Norway and the United Kingdom - with territorial claims, sometimes overlapping. Other countries
do not recognize any claims. The US and Russia maintain a “basis of claim”. All positions are explicitly
protected in Article IV, which preserves the status quo.
Subject) IR

Q.42)
Ans) b
Explanation) Option (b) is the correct answer.
Office of Profit mainly aims at ensuring separation of power between executive and legislature. Thus
statement b is correct. In Parliamentary democracy it is the duty of the legislature to keep executive
under check. If legislatures are appointed in various post then this function will be diluted. It is to maintain
independence of legislature and not of executive. Thus statement (c) is incorrect.
Statement (a) and (d) are incorrect as it has nothing to do with limited post nor with workload.
Subject) Polity

Q.43)
Ans) d
Source: http://pib.nic.in/newsite/PrintRelease.aspx?relid=181148
Explanation) Option (d) is the correct answer.
The issue of Ethics, Integrity and probity in public life have been a matter of concern all over the
government sector. In this context it is also extremely important that all railway servants adhere to
impeccable conduct and integrity at all times. “Mission Satyanishtha” launched on 27 th July 2018 aims at
sensitizing all railway employees about the need to adhere to good ethics and to maintain high standards
of integrity at work.
The objectives of the Mission are:
1. To train every employee to understand the need and value of ethics in Personal and Public life.
2. To deal with ethical dilemmas in life and Public Governance.
3. To help understand the policies of Indian Railways on ethics and integrity and the employee’s
role in upholding the same.
4. To develop inner governance through tapping inner resources.
Subject) Indian Economy

Q.44)
Ans) c
ForumIAS Offline
2nd Floor, IAPL House, 19, Pusa Road, Karol Bagh, New Delhi – 110005| helpdesk@forumias.academy
 
PTS 2019 | Simulator Test 6 (Set A) - Solutions | ForumIAS
 
Source: https://www.britannica.com/science/sound-physics
https://www.physicsclassroom.com/class/sound/Lesson-2/Pitch-and-Frequency
Explanation) Option (c) is the correct answer.
Option (a) is correct. Sound, a mechanical disturbance from a state of equilibrium that propagates
through an elastic material medium. When you bang a drum, its skin vibrates. The harder you bang, the
bigger the vibrations. The vibrating drum skin causes nearby air particles to vibrate, which in turn causes
other nearby air particles to vibrate. These vibrating particles make up a sound wave.
Option (b) and (d) are correct. Sound waves travel at 343 m/s through the air and faster through liquids
and solids. The waves transfer energy from the source of the sound, e.g. a drum, to its surroundings.
Your ear detects sound waves when vibrating air particles cause your ear drum to vibrate. The bigger the
vibrations the louder the sound. Hard surfaces reflect sound well, making echoes. Soft surfaces, like
curtains and carpets, reflect very little sound. They absorb the sound instead, so there are no echoes.
Option (c) is incorrect. The sensation of a frequency is commonly referred to as the pitch of a sound. A
high pitch sound corresponds to a high frequency sound wave and a low pitch sound corresponds to a
low frequency sound wave. The ability of humans to perceive pitch is associated with the frequency of
the sound wave that impinges upon the ear. Because sound waves traveling through air are longitudinal
waves that produce high- and low-pressure disturbances of the particles of the air at a given frequency,
the ear has an ability to detect such frequencies and associate them with the pitch of the sound.
Subject) Science

Q.45)
Ans) a
Relevance (Why the question has been asked): UPSC usually focuses on schemes and programs and
in S&T defence scheme stays a relevant subject to be covered.
Source: http://pib.nic.in/newsite/printrelease.aspx?relid=186966
http://makeinindiadefence.gov.in/pages/innovations-for-defence-excellence-idex-
Explanation) Option (a) – 1 and 2 only is the correct answer.
Statement 1 is correct. Innovations for Defence Excellence (iDEX) a scheme of Department of Defence
Production (DDP), formulated in collaboration with Start Up India and Atal Innovation Mission, with the
objective of bringing Start-ups to solve problems of defence production, innovate new technologies
required by defence forces and reduce dependence on imported technology was launched by the PM
during DefExpo 2018 on 12.04.2018. iDEX is aimed at creation of an ecosystem to foster innovation and
technology development in Defence and Aerospace by engaging Industries including MSMEs, Start-ups,
Individual Innovators, R&D institutes and Academia and provide them grants/funding and other support
to carry out R&D.
Statement 2 is correct. Innovations for Defence Excellence (iDEX) launched by the Government in April
2018, primarily aims at creation of an ecosystem to foster innovation and technology development in
Defence and Aerospace by engaging Industries including MSMEs, start-ups, individual innovators, R&D
institutes & academia, and provide them grants/funding and other support to carry out R&D which has
good potential for future adoption for Indian defence and aerospace needs. Key Functions of iDex:

ForumIAS Offline
2nd Floor, IAPL House, 19, Pusa Road, Karol Bagh, New Delhi – 110005| helpdesk@forumias.academy
 
PTS 2019 | Simulator Test 6 (Set A) - Solutions | ForumIAS
 
 Co-Innovation/co-creation
 Piloting of candidate technologies in important platforms
 Indigenization of various defence and aerospace related platforms being manufactured in the
country based on ToT.
Statement 3 is incorrect. iDEX will be funded and managed by a ‘Defence Innovation Organization
(DIO)’ which has been formed as a ‘not for profit’ company as per Section 8 of the Companies Act 2013
for this purpose, by the two founder members i.e. Defence Public Sector Undertakings (DPSUs) - HAL &
BEL.iDEX will function as the executive arm of DIO, carrying out all the required activities while DIO will
provide high level policy guidance to iDEX.
Subject) Technology

Q.46)
Ans) b
Source: https://www.thebetterindia.com/130232/constitutional-amendments-india-constitution/
Explanation) Option (b) is the correct answer.
Statement a is incorrect: The 61st Amendment of the Constitution of India, officially known as The
Constitution (Sixty-first Amendment) Act, 1988, lowered the voting age of elections to the Lok Sabha and
to the Legislative Assemblies of the States from 21 years to 18 years.
Statement b is correct: Constitution 52nd Amendment Act, 1985 provided provisions related to anti-
defection in India. In this amendment 10th schedule is added in the constitution
Statement c is incorrect: Special status to Delhi was granted through 69th Amendment Act
Statement d is incorrect: Free and compulsory education was added through 86th Amendment
Act and in this Article 21A was inserted in the constitution.
Subject) Polity

Q.47)
Ans) d
Relevance (Why the question has been asked):
Source:
NCERT
Explanation)
Subject) Polity

Q.48)
Ans) d
Source: DD BASU, Laxmikant
Explanation) Option (d) – 1, 2 and 3 is the correct answer.
Statement 1 is correct - Preamble - WE, THE PEOPLE OF INDIA, having solemnly resolved to
constitute India into a SOVEREIGN SOCIALIST SECULAR DEMOCRATIC REPUBLIC and to secure to
all its citizens...
ForumIAS Offline
2nd Floor, IAPL House, 19, Pusa Road, Karol Bagh, New Delhi – 110005| helpdesk@forumias.academy
 
PTS 2019 | Simulator Test 6 (Set A) - Solutions | ForumIAS
 
Statement 3 is correct - Article 51A - To promote harmony and the spirit of common brotherhood
amongst all the people of India transcending religious, linguistic and regional or sectional diversities; to
renounce practices derogatory to the dignity of women.
Statement 4 is correct - Articles 25-28 under Part III of the Constitution of India reflect its secular
character.
Subject) Polity

Q.49)
Ans) a
Relevance (Why the question has been asked):
Source: Atlas
Explanation)
Subject) Indian Geography

Q.50)
Ans) c
Relevance (Why the question has been asked):
Source: Laxmikant,
http://shodhganga.inflibnet.ac.in/bitstream/10603/117031/7/07_chapter%205.pdf
Explanation) Option (d) – 2 and 3 only is the correct answer.
Statement 1 is correct - According to the article 75, the Prime Minister shall be appointed by the
President and the other Ministers shall be appointed by the President on the advice of the Prime Minister.
Statement 2 is incorrect - The salaries and allowances of ministers shall be determined by the
Parliament.
Statement 3 is incorrect - In 1971, the Supreme Court (U.N. R. Rao vs Smt. Indira Gandhi) held that
dissolution of the House does not require that the Prime Minister and other ministers must resign, or
cease to hold office or must be dismissed by the President, because, Art. 74(1) is mandatory and the
President cannot exercise his executive power without the aid and advice of the Council of Ministers,
with the Prime Minister at the head.
Subject) Polity

Q.51)
Ans) b
Relevance (Why the question has been asked): There have been articles on decentralization and
grass root democracy like ‘Unorganised, decentralized India’, ‘No substitute for local governance’ which
speaks of the Uttaramerur inscription.
Source: NCERT Social Science Class 7: Chapter 2: New Kings and Kingdoms
http://www.newindianexpress.com/opinions/2019/apr/02/unorganised-decentralised-india-1958933.html
Explanation) Option (b) – Cholas is the correct answer.

ForumIAS Offline
2nd Floor, IAPL House, 19, Pusa Road, Karol Bagh, New Delhi – 110005| helpdesk@forumias.academy
 
PTS 2019 | Simulator Test 6 (Set A) - Solutions | ForumIAS
 
Uttaramerur inscription provides details about the administration of the Chola empire. It mentions about
the formation and functions of village councils, the method of revenue collection, the local self-
government, prescribed qualifications and disqualifications of members of the gram sabha, etc.
Subject) Medieval History

Q.52)
Ans) c
Source: Page 269 Shankar IAS (6th edition)
Explanation) The Global Climate Change Alliance (GCCA) is an initiative of the European Union. Its
overall objective is to build a new alliance on climate change between the European Union and the poor
developing countries that are most affected and that have the least capacity to deal with climate change.
The GCCA also provides technical and financial support to partner countries to integrate climate change
into their development policies and budgets, and to implement projects that address climate change on
the ground, promoting climate-resilient, low-emission development.
Limiting the global warming below 1.5 C is not among its direct aims.
Subject) Env - International Organisations

Q.53)
Ans) b
Relevance (Why the question has been asked): There was news regarding entry of women in a
Zoroastrian Fire Temple. Also, a fire temple Bhandara Atash Kadeh temple at 8787 Airport was opened
in North America
Source: Spectrum Facets of Indian Culture: Chapter 2; Religion and Philospohy
Explanation) Option (b) – 2 only is the correct answer.
Statement 2 is correct. Zoroastrianism is one of the oldest religions in the world founded by Iranian
prophet and reformer Zoroaster or Zarathustra. He rejected the prevailing religion of the Bronze Age
Persians because of its class structure and many gods, hence emphasized on monotheism. Water and
fire are considered elements of ritual purity in this faith. A Zoroastrian place of worship is called a Fire
Temple (Atash Bahram).
Statement 1 is incorrect. The holy book of Zoroastrianism is called the Avesta which contains the
Gathas (sacred songs). Torah is the book of Jews which contains the teachings and laws of Judaism.
Subject) Ancient History

Q.54)
Ans) b
Source: World Atlas
Explanation) Suez Canal is an artificial sea-level waterway located in Egypt and connects the
Mediterranean Sea with the Gulf of Suez, a northern branch of the Red Sea.

ForumIAS Offline
2nd Floor, IAPL House, 19, Pusa Road, Karol Bagh, New Delhi – 110005| helpdesk@forumias.academy
 
PTS 2019 | Simulator Test 6 (Set A) - Solutions | ForumIAS
 
The Bosporus or Bosphorus is a narrow, natural strait and an internationally significant waterway located
in northwestern Turkey. The Bosporus connects the Black Sea with the Sea of Marmara, and, by
extension via the Dardanelles, the Aegean and Mediterranean seas.
The Bab-el-Mandeb is a strait located between Yemen on the Arabian Peninsula, and Djibouti and Eritrea
in the Horn of Africa. It connects the Red Sea to the Gulf of Aden.
Subject) Geo - Mapping

Q.55)
Ans) c
Source: http://pib.nic.in/newsite/PrintRelease.aspx?relid=189634
http://www.newindianexpress.com/business/2018/jul/04/central-board-of-direct-taxes-signs-three-more-
unilateral-advance-pricing-agreements-with-taxpayers-1838078.html
https://www.pwc.in/assets/pdfs/publications/2014/advance-pricing-arrangements.pdf
https://www.indianeconomy.net/splclassroom/what-is-advanced-pricing-agreement/
Explanation) Option (c) – 1 and 2 is the correct answer.
Statement 1 is correct: Advance Pricing Agreement (APA) is a prospective agreement between a
taxpayer and a tax authority, determining the transfer pricing methodology and liabilities in case of
taxpayers’ international transactions to avoid future disputes. The methodology is to be applied for a
certain period of time based on the fulfilment of certain terms and conditions (called critical assumptions).
Statement 2 is correct: APAs gives certainty to taxpayers, reduce disputes, enhance tax revenues and
make the country an attractive destination for foreign investments. These agreements would be binding
on both the taxpayer as well as the government. Similarly, they lowers complaints and litigation costs.
What are the different types of APAs? An APA can be unilateral, bilateral, or multilateral.
• Unilateral APA: an APA that involves only the taxpayer and the tax authority of the country where
the taxpayer is located.
• Bilateral APA (BAPA): an APA that involves the taxpayer, associated enterprise (AE) of the
taxpayer in the foreign country, tax authority of the country where the taxpayer is located, and
the foreign tax authority.
• Multilateral APA (MAPA): an APA that involves the taxpayer, two or more AEs of the tax payer
in different foreign countries, tax authority of the country where the taxpayer is located, and the
tax authorities of AEs.
Subject) Indian Economy

Q.56)
Ans) b
Relevance (Why the question has been asked): Satellite launches, and spectrum allocations are quite
often mentioned in news. Thus, an aspirant must be aware of international governance and coordination
among nations in respect of these.
Source: https://www.itu.int/en/about/Pages/default.aspx
https://www.itu.int/en/about/Pages/vision.aspx

ForumIAS Offline
2nd Floor, IAPL House, 19, Pusa Road, Karol Bagh, New Delhi – 110005| helpdesk@forumias.academy
 
PTS 2019 | Simulator Test 6 (Set A) - Solutions | ForumIAS
 
https://en.wikipedia.org/wiki/International_Telecommunication_Union
Explanation) Option (b) is the correct answer.
Option (a) and (c) is correct. Originally founded in 1865 to promote cooperation among international
telegraphy networks of the day, ITU predates many other standardization bodies and its long and
distinguished history contains a number of important 'firsts', such as the standardization of the use of the
Morse code and the world's first radio communication and fixed telecommunication networks. It is
headquartered in Geneva. Also, it is the oldest of the 15 Specialized agencies of the United Nations.
Option (b) is incorrect. ITU embodies principles of public-private partnership, with its current
membership of 193 countries and over 800 private-sector entities and academic institutions. ITU
membership represents a cross-section of the global ICT sector, from the world's largest vendors,
manufacturers and telecom operators to small, innovative players and SMEs working with new and
emerging technologies, along with leading R&D institutions and academia.
Option (d) is correct. ITU allocates global radio spectrum and satellite orbits, develops the technical
standards that ensure networks and technologies seamlessly interconnect, and strives to improve access
to ICTs to underserved communities worldwide. Every time you make a phone call via the mobile, access
the Internet or send an email, you are benefitting from the work of ITU.
Subject) IR

Q.57)
Ans) d
Relevance (Why the question has been asked):
Source: Laxmikant, DD BASU,
https://www.constitution.org/cons/india/p05101.html
https://www.sci.gov.in/supremecourt/2016/12704/12704_2016_Judgement_26-Sep-2018.pdf
Explanation)
Subject) Polity

Q.58)
Ans) d
Source: Page 14, 17, 31-32 (M. Barnwal)
Explanation) The Indus and Dihang Gorges are found on the north-western and north-eastern ends of
the Himalayas. These syntaxial bends caused due to the pressure, give an impression that the mountain
ranges beyond the gorges are also included in the Himalayas as if ranges are bending along an axis.
Therefore, these gorges are not the ends of the Himalayas, but are part of the mountain ranges beyond
the Indus Gorge, like Sulaiman, Kirthar etc and the ranges beyond the Dihang Gorge like Arakanyoma
and the mountains of north-east region.
Western Ghats extends from the mouth of the river Tapi to the Cape of Kanyakumari for a distance of
1600 km.
Pulicat Lake is the second largest brackish water lagoon in India, after Chilika Lake. It is located at the
boundary of Andhra Pradesh and Tamiln Nadu. Major part of the lake comes under Nellore district of

ForumIAS Offline
2nd Floor, IAPL House, 19, Pusa Road, Karol Bagh, New Delhi – 110005| helpdesk@forumias.academy
 
PTS 2019 | Simulator Test 6 (Set A) - Solutions | ForumIAS
 
Andhra pradesh. The lake encompasses the Pulicat Lake Bird Sanctuary. The barrier island of
Sriharikotaseparates the lake from the Bay of Bengal and is home to the Satish Dhawan Space Centre.
The lake located between Krishna and Godawari deltas is the Kolleru lake.
Subject) Indian Geography

Q.59)
Ans) b
Source: 63, Shankar Academy
Explanation) Biological Oxygen Demand is the amount of dissolved oxygen needed by bacteria in
decomposing the organic wastes present in the water. It is directly related to increasing input of organic
wastes. Therefore, it increases with the increase in the amount of organic waste.
Higher value of BOD implies higher Oxygen consumption for for waste decomposition, which decreases
the DO content of the water
Chemical Oxygen Demand is the measure of oxygen equivalent of the requirement of oxidation of total
organic matter, i.e. biodegradable and non-biodegrdable, present in water. Thus, COD is inclusive of
BOD and BOD can never be greater than COD
Subject) Ecology

Q.60)
Ans) d
Source: Shankar IAS Environment Page #235
http://www.pib.nic.in/newsite/erelcontent.aspx?relid=57051
Explanation) Option (d) – 1, 2 and 3 is the correct answer.
The basic objectives that should govern the National Forest Policy- are the following:
 Maintenance of environmental stability through preservation and, where necessary, restoration
of the ecological balance that has been adversely disturbed by serous depletion of the forests of
the country.
 Conserving the natural heritage of the country by preserving the remaining natural forests with
the vast variety of flora and fauna, which represent the remarkable biological diversity and genetic
resources of the country.
 Checking soil erosion and denudation in the catchments areas of rivers, lakes, reservoirs in the
“interest of soil and water conservation, for mitigating floods and droughts and for the retardation
of siltation of reservoirs.
 Checking the extension of sand-dunes in the desert areas of Rajasthan and along the coastal
tracts.
 Increasing substantially the forest/tree cover in the country through massive afforestation and
social forestry programmes, especially on all denuded, degraded and unproductive lands.
 Meeting the requirements of fuel-wood, fodder, minor forest produce and small timber of the rural
and tribal populations.
 Increasing the productivity of forests to meet essential national needs.
ForumIAS Offline
2nd Floor, IAPL House, 19, Pusa Road, Karol Bagh, New Delhi – 110005| helpdesk@forumias.academy
 
PTS 2019 | Simulator Test 6 (Set A) - Solutions | ForumIAS
 
 Encouraging efficient utilisation of forest produce and maximising substitution of wood.
 Creating a massive people’s movement with the involvement of women, for achieving these
objectives and to minimise pressure on existing forests.
Subject) Env - International Organisations

Q.61)
Ans) c
Relevance (Why the question has been asked): Recently in the news, Gender Based Violence (GBV)
is a global health, human rights and development issue that has affected every community and country
in every corner of the world. In India, gender-based violence has many manifestations like rape, dowry,
honour killings, acid attacks, witch - hunting, sexual harassment, child sexual abuse, child marriage, sex
selective abortion etc. which makes it probable topic from which UPSC can ask questions.
Source: https://www.indiatoday.in/education-today/gk-current-affairs/story/-234-one-stop-centres-
opened-for-women-here-is-how-to-access-1410015-2018-12-15
http://vikaspedia.in/social-welfare/women-and-child-development/women-development-1/poorna-shakti-
kendras
https://www.hindustantimes.com/india-news/government-to-set-up-over-500-more-one-stop-centres-for-
women-affected-by-violence/story-799ViqFWLhoxPWyAWKXToL.html
Explanation) Option (c) – Both 1 and 2 is the correct answer.
Statement 1 is correct. Ministry of Women and Child Development (MWCD), has formulated a Centrally
Sponsored Scheme for setting up One Stop Centre. These centres are intended to support women
affected by violence in private and public spaces within the family, community and at the workplace.
Women facing physical, sexual, emotional, psychological and economic abuse, irrespective of age, class,
caste, education status, marital status, race and culture will be facilitated with support and redressal.
These Centres are being established across the country to provide integrated support and assistance
under one roof to women affected by violence, both in private and public spaces in phased manner.
Target group: The OSC will support all women including girls below 18 years of age affected by violence,
irrespective of caste, class, religion, region, sexual orientation or marital status
Statement 2 is correct. The Scheme will be funded through Nirbhaya Fund. The Central Government
will provide 100% financial assistance to the State Government /UT Administrations under the Scheme.
The day to day implementation and administrative matters would be the responsibility of the State
Government.
Subject) Social Schemes

Q.62)
Ans) b
Source: NCERT Social Science Class 8: Chapter 2: From trade to territory
Explanation) Option (b) is the correct answer.
Dual Government in Bengal was introduced by Robert Clive in 1765 and continued till 1772 (abolished
by Warren Hastings)
ForumIAS Offline
2nd Floor, IAPL House, 19, Pusa Road, Karol Bagh, New Delhi – 110005| helpdesk@forumias.academy
 
PTS 2019 | Simulator Test 6 (Set A) - Solutions | ForumIAS
 
Under this system the administration of Bengal was divided in two parts- Diwani and Nijamat. Diwani
meant the right to collect revenue which was entrusted to the company. Nijamat meant administrative
and territorial rights and these were given to the Nawab. This was known as the system of dual or double
government.
Subject) Modern History

Q.63)
Ans) c
Relevance (Why the question has been asked):Recently India and OECD(Organization for Economic
Cooperation and Development) sign agreement to enable India’s participation in Programme for
International Students Assessment (PISA) to be held in 2021 also participation in PISA will improve
learning levels of children and enhance the quality of education in the country and learnings from
participation in PISA will help to introduce competency based examination reforms in the school system
and help move away from rote learning.
Source: http://pib.nic.in/newsite/PrintRelease.aspx?relid=187830
https://www.thebetterindia.com/158849/india-pisa-news-students/
Explanation) Option (c) – 3 only is the correct answer.
Statement 1 is incorrect. PISA is a competency-based assessment which unlike content-based
assessment, measures the extent to which students have acquired key competencies that are essential
for full participation in modern societies. It would lead to recognition and acceptability of Indian students
and prepare them for the global economy in the 21st century.
PISA measures student performance in mathematics, reading and science and even innovative subjects
like collaborative problem-solving and money literacy. The test is designed by education experts around
the world. The assessments do not depend on mugging up of dates or theories but assess how students
apply what they have learned to real-world problems. Countries willing to participate choose kids from
different regions to represent a sample of 15-year-olds in their country.
Statement 2 is incorrect. In India, schools run by Kendriya Vidyalaya Sangathan (KVS), Navodaya
Vidyalaya Samiti (NVS) and schools in the UT of Chandigarh will participate.
Statement 3 is correct. It is conducted by the Organization for Economic Cooperation and Development
(OECD).
Subject) Social Schemes

Q.64)
Ans) d
Source: http://pib.nic.in/newsite/PrintRelease.aspx?relid=189032
Explanation) Option (d) is the correct answer
The first report on EASE Reforms Index that independently measures progress on the Public Sector
Banks (PSB) Reforms Agenda was announced last January on 140 objective, benchmarked metrics. The
publishing of the independent report was commissioned through Indian Banks’ Association.
Subject) Indian Economy

ForumIAS Offline
2nd Floor, IAPL House, 19, Pusa Road, Karol Bagh, New Delhi – 110005| helpdesk@forumias.academy
 
PTS 2019 | Simulator Test 6 (Set A) - Solutions | ForumIAS
 

Q.65)
Ans) b
Source: http://ccacoalition.org/en/content/about
Explanation) The Climate and Clean Air Coalition to Reduce Short-Lived Climate Pollutants (CCAC) was
launched by the United Nations Environment Programme (UNEP) and six countries — Bangladesh,
Canada, Ghana, Mexico, Sweden, and the United States — on 16 February 2012. The CCAC aims to
catalyze rapid reductions in short-lived climate pollutants to protect human health, agriculture and the
environment.
Mitigating carbon dioxide emission is not an objective of CCAC, rather its objective is complementary to
mitigating CO2 emissions.
Subject) Env - International Organisations

Q.66)
Ans) c
Source: 128, Khullar
Explanation) The Indian Ocean Dipole (IOD) is defined as the difference in sea surface temperature
between two areas (or poles, hence a dipole) – a western pole in the Arabian Sea (western Indian Ocean)
and an eastern pole in the eastern Indian Ocean south of Indonesia. A positive IOD occurs when the sea
surface temperatures are greater than normal in the Arabian Sea and less than normal in the tropical
eastern Indian Ocean. When the reverse is the case, a negative IOD is said to have developed.
A positive IOD leads to greater monsoon rainfall and more active (above normal rainfall) monsoon days
while negative IOD leads to less rainfall and more monsoon break days
Subject) World Geography

Q.67)
Ans) a
Source: https://www.iucn.org/asia/countries/bangladesh/gharial-conservation
Explanation) Gharial is placed in the Critically Endangered category of the Red List of IUCN. Its habitat
include terrestrial habitat and fresh water. Chilika lake is a brackish water lake, which is not a suitable
habitat for gharial. Presently, the wild populations of gharials can only be found in Bangladesh, India and
Nepal. The gharial is distributed in the river Ganga and the Brahmaputra river and some of their
tributaries. It is also found in rivers like Chambal, Son etc
Subject) Biodiversity

Q.68)
Ans) d
Relevance (Why the question has been asked): Recently after the Pulwama attack, it was the first time
that India has been invited to a meeting of the OIC. It was very much prominent event covered by almost
all news agencies.

ForumIAS Offline
2nd Floor, IAPL House, 19, Pusa Road, Karol Bagh, New Delhi – 110005| helpdesk@forumias.academy
 
PTS 2019 | Simulator Test 6 (Set A) - Solutions | ForumIAS
 
Source: https://www.indiatoday.in/education-today/gk-current-affairs/story/oic-2019-islamic-summit-
india-invited-first-time-sushma-swaraj-1468689-2019-03-02
https://www.oic-oci.org/page/?p_id=52&p_ref=26&lan=en
https://en.wikipedia.org/wiki/Member_states_of_the_Organisation_of_Islamic_Cooperation
Explanation) Option (d) – Neither 1 nor 2 is the correct answer.
Statement 1 is incorrect. The Organization of Islamic Cooperation (OIC), founded in 1969, is the second
largest intergovernmental organization after the United Nations with a membership of 57 states spread
over four continents. The Organization is the ‘collective voice of the Muslim world’. It endeavours to
safeguard and protect the interests of the Muslim world in the spirit of promoting international peace and
harmony among various people of the world.
The Organization was established upon a decision of the historical summit which took place in Rabat,
Kingdom of Morocco on 12th Rajab 1389 Hijra (25 September 1969) following the criminal arson of Al-
Aqsa Mosque in occupied Jerusalem.

Statement 2 is incorrect. The Organisation of Islamic Cooperation has 57 members, 56 of which are
also member states of the United Nations with 47 countries being Muslim majority countries. Some,
especially in West Africa, are – though with large Muslim populations – not necessarily Muslim majority
countries. A few countries with significant Muslim populations, such as Russia and Thailand, sit as
Observer States, while others, such as India and Ethiopia, are not members.

Subject) IR

Q.69)
Ans) a
Source: An Introduction to Indian Art: Class XI: Chapter 6: Temple architecture and sculpture, pg no 73
Explanation) Option (a) is the correct answer.
The panel of Ganjendramoksha is depicted on the walls of the Dashavatara Temple, an early 6th century
Vishnu Hindu temple located at Deogarh, Uttar Pradesh.
Subject) Art and Culture

Q.70)
Ans) d
Relevance (Why the question has been asked): Channapatna toys were in news due to loss of
livelihood of these people. There was also news about Bengal’s kantha to be spun in Delhi to revive the
dying art.
Source: https://www.thehindu.com/elections/lok-sabha-2019/a-poll-battle-in-the-land-of-silk-and-
toys/article26848325.ece
Explanation) Option (d) – 1, 2 and 3 is the correct answer.
Pair 1 is correctly matched. Kantha is an ancient embroidery style popular in the subcontinent's eastern
region including West Bengal.

ForumIAS Offline
2nd Floor, IAPL House, 19, Pusa Road, Karol Bagh, New Delhi – 110005| helpdesk@forumias.academy
 
PTS 2019 | Simulator Test 6 (Set A) - Solutions | ForumIAS
 
Pair 2 is correctly matched. Channapatna toys are the wooden toys that get their name from the town
of Channapatna (Gombegala ooru) in Ramanagar district in the state of Karnataka.
Pair 3 is correctly matched. Pochampally Saree or Pochampalli Ikat is a saree made in Bhoodan
Pochampally, Nalgonda district, Telanagana, India.
Subject) Art and Culture

Q.71)
Ans) b
Source: Page: 560-570 DR Khullar
Explanation) Rainwater harvesting through recharge pit is suitable for alluvial areas where permeable
rocks are exposed on the land surface or at very shallow depth. This technique is used for harvesting
roof-top rainwater to recharge the shallow aquifers. In areas with deeper aquifers roof-top rainwater
harvesting is done through the existing tubewells that are tapping these deeper aquifers.
Subject) Pollution and Climate Change

Q.72)
Ans) c
Relevance (Why the question has been asked): Recently in the news , Bhasha Sangam initiative under
the ‘Ek Bharat Shreshtha Bharat’ aims to make the students aware about the unique cultural, ethnic and
linguistic diversity of our country .
Source: http://pib.nic.in/newsite/PrintRelease.aspx?relid=186496
http://vikaspedia.in/education/education-best-practices/201cbhasha-sangam201d-initiative-for-class-10-
and-12-students
https://www.indiatoday.in/education-today/news/story/prakash-javadekar-indian-languages-1396279-
2018-11-26
Explanation) Option (c) – 2 and 3 only is the correct answer.
Statement 1 is incorrect. The Objective of Bhasha Sangam Programme are
● To introduce school students to all the 22 Indian Languages of Schedule VIII of the Constitution
of India.
● To enhance linguistic tolerance and respect, and promote national integration.
While to assess the academic performance of students and ensure improvement in quality of
education in primary and upper primary schools is objective of Prathiba Parv and not of Bhasha
Sangam Programme.
Statement 2 is correct. Bhasha Sangam Programme provides an opportunity to schools and educational
institutions (BIETS, DIETs, CTEs/IASEs, SCERTs, SIEs, School Boards, Directorates of School
Education, etc.) to provide multilingual exposure to students in Indian Languages. The objective is to
familiarize every child with simple dialogues in all the 22 languages under Schedule VIII of the
Constitution of India, taking up one language on each working day, to enhance linguistic tolerance and
promote national integration.
Features of Bhasha Sangam Programme:

ForumIAS Offline
2nd Floor, IAPL House, 19, Pusa Road, Karol Bagh, New Delhi – 110005| helpdesk@forumias.academy
 
PTS 2019 | Simulator Test 6 (Set A) - Solutions | ForumIAS
 
● This will be run by the State/UT Department of School Education.
● A short dialogue consisting of five simple commonly used sentences has been designed in 22
languages for use by students of all classes.
● Sentences from each language are to be shared with student’s everyday for 22 working days and
everyday only one language will be shared.
● A digital book with audio recordings of the dialogue is available on the websites
http://epathshala.gov.in/ so that students can hear the correct pronunciation. A mobile app is
also made available to access the e-resources.
● The suggested activities should be carried out in a joyful and interesting manner. Students have
enjoyed the activities and participated with great enthusiasm wherever this initiative has been
piloted. This initiative is not mandatory and there should be no formal testing of any kind.
● Heads of schools may upload photographs and videos of daily activities under Bhasha Sangam.
Statement 3 is correct. The Bhasha Sangam initiative under the ‘Ek Bharat Shreshtha Bharat’ aims to
make the students aware about the unique cultural, ethnic and linguistic diversity of our country.
Subject) Social Schemes

Q.73)
Ans) c
Relevance (Why the question has been asked): Aspirants are expected to be aware of Basic Chemical
names, compositions, uses of such important and significant gases.
Source: https://www.netl.doe.gov/research/coal/energy-systems/gasification/gasifipedia/syngas-
composition
https://en.wikipedia.org/wiki/Syngas
https://en.wikipedia.org/wiki/Nitrous_oxide
https://en.wikipedia.org/wiki/Noble_gas
Explanation) Option (c) – 1 and 3 only is the correct answer.
Pair 1 is correct. Syngas, or synthesis gas, is a fuel gas mixture consisting primarily of hydrogen, carbon
monoxide, and very often some carbon dioxide. The name comes from its use as intermediates in creating
synthetic natural gas (SNG)[1] and for producing ammonia or methanol. Syngas is usually a product of
gasification and the main application is electricity generation. Syngas is combustible and often used as a
fuel for internal combustion engines. It has less than half the energy density of natural gas.

Pair 2 is incorrect. Nitrous oxide, commonly known as laughing gas or nitrous, is a chemical compound,
an oxide of nitrogen with the formula N 2O. At room temperature, it is a colourless non-flammable gas,
with a slight metallic scent and taste. At elevated temperatures, nitrous oxide is a powerful oxidiser similar
to molecular oxygen. It is soluble in water. Nitrous oxide has significant medical uses, especially in
surgery and dentistry for its anaesthetic and pain reducing effects. Its name "laughing gas", coined by
Humphry Davy, is due to the euphoric effects upon inhaling it, a property that has led to its recreational
use as a dissociative anaesthetic. It is on the World Health Organization's List of Essential Medicines,
the most effective and safe medicines needed in a health system. It also is used as an oxidiser in rocket
propellants, and in motor racing to increase the power output of engines.
ForumIAS Offline
2nd Floor, IAPL House, 19, Pusa Road, Karol Bagh, New Delhi – 110005| helpdesk@forumias.academy
 
PTS 2019 | Simulator Test 6 (Set A) - Solutions | ForumIAS
 
Pair 3 is correct. The noble gases (historically also the inert gases; sometimes referred to as aerogens)
make up a group of chemical elements with similar properties; under standard conditions, they are all
odorless, colorless, monatomic gases with very low chemical reactivity. The six noble gases that occur
naturally are helium (He), neon (Ne), argon (Ar), krypton (Kr), xenon (Xe), and the radioactive radon (Rn).
Oganesson (Og) is variously predicted to be a noble gas as well or to break the trend due to relativistic
effects; its chemistry has not yet been investigated.
Subject) Science

Q.74)
Ans) a
Relevance (Why the question has been asked): IONS was in the news as it completed ten years of its
existence. This becomes prominent for both strategic and technological cooperation.
Source: http://www.ions.global/
https://www.indiatoday.in/education-today/gk-current-affairs/story/ions-indian-ocean-naval-symposium-
1387711-2018-11-14

Explanation) Option (a) – 1 and 2 only is the correct answer.


Statement 1 is correct. The ‘Indian Ocean Naval Symposium’ (IONS) is a voluntary initiative that seeks
to increase maritime co-operation among the navies of the littoral states of the Indian Ocean Region by
providing an open and inclusive forum for discussion of regionally relevant maritime issues. In the
process, it endeavours to generate a flow of information between naval professionals that would lead to
common understanding and possibly cooperative solutions on the way ahead. The launch of so important
a regional initiative was able to meet with such wide acceptance across the length and breadth of the
Indian Ocean was in itself a unique phenomenon. The IONS act as a security construct for the Indian
Ocean region and apart from its series of symposiums, it conducts numerous other activities like
workshops, essay competitions and lectures to promote its objective.
Statement 2 is correct. It was first launched in 2008 and its biennial meetings among the littoral states
of the Indian Ocean region have been held ever since. In April 2018 the IONS Chairmanship rotated to
Iran, and in 2020 would rotate to France.

Statement 3 is incorrect. The organization is celebrated 2018 as the 10th anniversary of IONS with the
theme 'IONS as a Catalyst for SAGAR' (where SAGAR stands for 'Security and Growth for All in the
Region'). It is in consonance with India's 'Act East' policy and the nation's diplomatic, economic and
military outreach in the region.

Subject) Technology

Q.75)
Ans) a
Source: An Introduction to Indian Art: Class XI: Chapter 4
Spectrum Facets of Indian Culture: Chapter 7
Explanation) Option (a) – 2 only is the correct answer.
ForumIAS Offline
2nd Floor, IAPL House, 19, Pusa Road, Karol Bagh, New Delhi – 110005| helpdesk@forumias.academy
 
PTS 2019 | Simulator Test 6 (Set A) - Solutions | ForumIAS
 
Cave architecture in ancient India was a product of the unique socio-religious conditions prevailing in that
period. Lomas Rishi is one of the caves of the group of caves at Barabari in Bihar. It was built during the
Mauryan period.
The Nasik caves, also referred to as Pandavleni Caves,are a group of 24 caves carved between the 1st
century BCE and the 3nd century CE, before the Gupta period.
Bagh caves are one of the most famous that were carved during the Gupta rule.
Subject) Art and Culture

Q.76)
Ans) b
Source: https://www.thehindu.com/business/Industry/potato-farmers-cry-foul-as-pepsico-sues-
them/article26936480.ece
Explanation) Option (b) – Both 1 and 2 is the correct answer.
Subject) Polity

Q.77)
Ans) c
Source: http://pib.nic.in/newsite/PrintRelease.aspx?relid=186804
Explanation) Option (c) – 3 and 4 only is the correct answer.
Statement 1 is incorrect:100% FDI under automatic route is permitted in marketplace model of e-
commerce.
Statement 2 is incorrect: FDI is not permitted in inventory-based model of e-commerce.
Statement 3 is correct: In marketplace model, any warrantee/ guarantee of goods and services sold will
be responsibility of the seller.
Statement 4 is correct: e-commerce marketplace entity cannot mandate any seller to sell any product
exclusively on its platform only.
Subject) Indian Economy

Q.78)
Ans) a
Relevance (Why the question has been asked): The term has become more prominent in recent times,
specifically after Mission Shakti led ASAT conducted by India.
Source: https://bigthink.com/paul-ratner/how-the-kessler-syndrome-can-end-all-space-exploration-and-
destroy-modern-life
https://www.spacesafetymagazine.com/space-debris/kessler-syndrome/
Explanation) Option (a) is the correct answer.
The Kessler Syndrome is a theory proposed by NASA scientist Donald J. Kessler in 1978, used to
describe a self-sustaining cascading collision of space debris in Low Earth Orbit (LEO). It’s the idea that
two colliding objects in space generate more debris that then collides with other objects, creating even

ForumIAS Offline
2nd Floor, IAPL House, 19, Pusa Road, Karol Bagh, New Delhi – 110005| helpdesk@forumias.academy
 
PTS 2019 | Simulator Test 6 (Set A) - Solutions | ForumIAS
 
more shrapnel and litter until the entirety of LEO is an impassable array of super swift stuff. At that point,
any entering satellite would face unprecedented risks of headfirst bombardment.
Subject) Technology

Q.79)
Ans) c
Source: Indian Economy by Shankar Ganesh Page# 164
Explanation) Option (c) – Both 1 and 2 is the correct answer.
Statement 1 is correct. As an exchange rate system, SDR is an average exchange rate derived from
basket of four currencies – Dollar, Euro, Pound and Yen. The exchange rate of a country is expressed
against SDR for example India’s quota is expressed as SDR 4158.2 million
Statement 2 is correct. As a loan arrangement, the member countries are entitled to get loan from IMF’s
Special Drawing Accounts. This loan amount is upto 200% of the member’s quota with IMF.
Subject) Indian Economy

Q.80)
Ans) d
Explanation) The Wildlife Institute of India developed the app, called the Monitoring System for Tiger-
Intensive Protection and Ecological Status or M-STrIPES). It is an software-based monitoring system
launched across Indian tiger reserves by the Indian government's National Tiger Conservation Authority
(NTCA) in 2010.
The first tiger census conducted in 1972, is every four years by the National Tiger Conservation
Authorities (NTCA). In 2018, NTCA used an android app named MSTrIPES for the proper location data
feeding and filling the record more accurately.
In 2019 NTCA issued an advisory to all the field directors making the application imperative even in all
tiger-bearing forests outside reserves. Earlier it was used only in the designated tiger reserves.
Subject) Biodiversity

Q.81)
Ans) d
Source: NITI Ayog new india @75- page 9
http://niti.gov.in/writereaddata/files/Strategy_for_New_India.pdf
https://www.business-standard.com/article/markets/india-s-debt-to-gdp-ratio-second-worst-among-
emerging-markets-report-119022400732_1.html
Explanation) Option (d) – 1, 2 and 3 is the correct answer.
The debt-to-GDP ratio is the ratio of a country's public debt to its gross domestic product (GDP). By
comparing what a country owes with what it produces, the debt-to-GDP ratio indicates its ability to pay
back its debts.
Statement 1 is correct: As debt decrease, interest payment liability of government will also decrease.

ForumIAS Offline
2nd Floor, IAPL House, 19, Pusa Road, Karol Bagh, New Delhi – 110005| helpdesk@forumias.academy
 
PTS 2019 | Simulator Test 6 (Set A) - Solutions | ForumIAS
 
Statement 2 is correct: At present India’s debt to GDP ratio is high compared to other emerging
economies. Thus reduction in debt will bring the size of India’s government debt closer to that of other
emerging market economies.
Statement 3 is correct: Decrease debt to gdp means government will take less from the market, thus
there will be availability of credit for the private sector in the financial markets
Subject) Indian Economy

Q.82)
Ans) c
Source: https://www.livemint.com/Politics/Zag39IyZPanr0mNFR0VFbL/France-to-introduce-tax-on-
global-internet-technology-firms.html
Explanation) Option (c) – France is the correct answer.
France has been pushing hard for a new so-called “GAFA tax" -- named after Google, Apple, Facebook
and Amazon -- to ensure the global giants pay a fair share of taxes on their massive business operations
in Europe.
Subject) Economics

Q.83)
Ans) a
Relevance (Why the question has been asked): The Union Cabinet has cleared a proposal to convert
the GST Network into a government-owned company.
Source: https://www.indianeconomy.net/splclassroom/what-is-goods-and-services-tax-network-gstn/
https://economictimes.indiatimes.com/news/economy/policy/cabinet-clears-proposal-to-convert-gstn-
into-government-entity/articleshow/65965553.cms
http://pib.nic.in/newsite/PrintRelease.aspx?relid=183709
Explanation) Option (a) – 1 and 2 only is the correct answer.
Launch of GST needed mega infrastructural support including IT facilities.
For this task, the government has created Goods and Services Tax Network (GSTN)
Statement 1 is correct: Earlier the Governments – centre, states plus UTs hold 49% of GSTN. But
recently GST Council approved the change in ownership by acquisition of entire 51% equity held by the
Non-Government Institutions in GSTN equally by the Centre and the State Governments
Statement 2 is correct: Central government holds 24.5% while the remaining governmental share of
24.5% is held by states and UTs.
Statement 3 is incorrect: It provides the IT backbone to the indirect tax regime because GST is indirect
tax.
Subject) Economics

Q.84)
Ans) c

ForumIAS Offline
2nd Floor, IAPL House, 19, Pusa Road, Karol Bagh, New Delhi – 110005| helpdesk@forumias.academy
 
PTS 2019 | Simulator Test 6 (Set A) - Solutions | ForumIAS
 
Relevance (Why the question has been asked): UPSC picks up terms and keywords generally used
for describing an incident/bilateral relation etc. Wuhan spirit has been in the news and the recent
controversy over 1267 Resolution questioned the achievements of the Wuhan summit.
Source: https://www.thehindu.com/news/international/china-backs-wuhan-spirit-despite-differences-on-
bri/article26885078.ece
https://www.deccanherald.com/opinion/second-edit/what-happened-wuhan-spirit-701963.html
https://economictimes.indiatimes.com/news/politics-and-nation/pm-narendra-modi-meets-chinese-
president-xi-on-sidelines-of-sco-summit/articleshow/64520120.cms
https://www.orfonline.org/tags/wuhan-summit/
Explanation) Option (c) is the correct answer.
Prime Minister Narendra Modi and Chinese President Xi Jinping met for an ‘informal summit’ at Wuhan,
the two sides agreed on steps to reset bilateral relations derailed by the Doklam crisis. The process of
rapprochement apparently set in motion at this summit, which came to be described as the ‘Wuhan spirit’,
was expected to guide their relations.
Subject) IR

Q.85)
Ans) d
Relevance (Why the question has been asked): Recently the National Green Tribunal (NGT) upheld
the environmental clearance granted to the India-based Neutrino Observatory (INO).
Source:
https://indianexpress.com/article/explained/neutrino-facility-neutrino-observatory-national-green-
tribunal-ino-5441579/
https://www.thehindu.com/sci-tech/science/what-are-neutrinos-and-how-are-they-
detected/article23546887.ece
Explanation) Option (d) – 1, 2 and 3 is the correct answer.
Statement 1 is correct. Neutrinos come in three types or “flavours” – electron neutrino, tau neutrino and
muon neutrino. They can change from one flavor to another as they travel. This process is called neutrino
oscillation and is an unusual quantum phenomenon. Neutrino oscillation was established by Sudbury
Neutrino Observatory, Canada, and Super-Kamiokande experiment in Japan. They studied Solar
neutrinos, atmospheric neutrinos and man-made neutrinos. The India-based Neutrino Observatory (INO)
will study atmospheric neutrinos only. Solar neutrinos have much lower energy than the detector can
detect.
Statement 2 is correct. Proton, neutron, and electrons are tiny particles that make up atoms. The
neutrino is also a tiny elementary particle, but it is not part of the atom. Such particles are also found to
exist in nature. Neutrino has a very tiny mass, no charge and spin half. Further neutrinos are neutral
particles which cannot be attracted by the magnets. It interacts very weakly with other matter particles.
So weakly that every second trillions of neutrinos fall on us and pass through our bodies unnoticed.
Statement 3 is correct. Neutrinos interact very little with the matter around them, so they travel long
distances uninterrupted. Since they take time to cross these distances, they are in effect uninterrupted

ForumIAS Offline
2nd Floor, IAPL House, 19, Pusa Road, Karol Bagh, New Delhi – 110005| helpdesk@forumias.academy
 
PTS 2019 | Simulator Test 6 (Set A) - Solutions | ForumIAS
 
for very long times. The extragalactic neutrinos we observe may be coming from the distant past. These
inviolate messengers can give us a clue about the origin of the universe and the early stages of the infant
universe, soon after the Big Bang.
Subject) Technology

Q.86)
Ans) a
Source: https://www.livelaw.in/right-choose-life-partner-fundamental-right-consent-family-community-
clan-not-necessary-marriage-two-adults-sc-read-judgment/
https://indiankanoon.org/doc/92846055/
Explanation) Option (a) is the correct answer.
According to the judgment of the supreme court in Shakti Vahini vs Union Of India (2018), When two
adults consensually choose each other as life partners, it is a manifestation of their choice which is
recognized under Articles 19 and 21 of the Constitution. The State is duty-bound to protect the
fundamental rights of its citizens; and an inherent aspect of Article 21 of the Constitution would be the
freedom of choice in marriage.
Subject) Polity

Q.87)
Ans) c
Relevance (Why the question has been asked): Recently in the news, It will help us in finding the
habitable planets around the most ubiquitous stars in our galaxy.
Source: https://indianexpress.com/article/technology/science/astrocomb-to-help-hunt-for-planets-alien-
life-5596481/
Explanation) Option (c) is the correct answer.
Scientists have developed an astrocomb, a tool that precisely measures frequencies or colours of light
which can help widen the search for Earth-like planets and perhaps extra-terrestrial life. The custom-
made frequency comb developed by researchers from the National Institute of Standards and Technology
(NIST) in the US, it provides the precision needed for discovering and characterising planets orbiting M
dwarf stars, which comprise 70 per cent of the stars in the galaxy and are plentiful near Earth. A star’s
nuclear furnace emits white light which is modified by elements in the atmosphere that absorb certain
narrow bands of colour.
Subject) Technology

Q.88)
Ans) c
Relevance (Why the question has been asked): Recently in the news, Since this flagship scheme of
the Central Government for employment generation has crossed the milestone of one crore beneficiaries
as on January 14, 2019 so it becomes important from UPSC perspective.
Source: http://pib.nic.in/newsite/PrintRelease.aspx?relid=187563

ForumIAS Offline
2nd Floor, IAPL House, 19, Pusa Road, Karol Bagh, New Delhi – 110005| helpdesk@forumias.academy
 
PTS 2019 | Simulator Test 6 (Set A) - Solutions | ForumIAS
 
http://vikaspedia.in/social-welfare/unorganised-sector-1/schemes-unorganised-sector/pradhan-mantri-
rojgar-protsahan-yojana
https://cleartax.in/s/pradhan-mantri-rojgar-protsahan-yojana
Explanation) Option (c) – 2 and 3 only is the correct answer.
Statement 1 is incorrect. The Pradhan Mantri Rojgar Protsahan Yojana (PMRPY) is a scheme to
incentivise employers registered with the Employees' Provident Fund Organisation for job creation by the
Government paying the 8.33% contribution of employers to the Employee Pension Scheme in respect of
new employees having a new Universal Account Number (UAN). For the textile (apparel) sector, the
Government will also be paying the 3.67% Employees Provident Fund (EPF) contribution of the eligible
employer for these new employees.
A direct benefit of this scheme is that these workers have access to social security benefit through
Provident Fund, Pension and Death Linked Insurance. The terminal date for registration of beneficiaries
through an establishment under the PMRPY is March 31, 2019
Statement 2 is correct. The entire system is online and AADHAR based with no human interface in the
implementation of the scheme.
Statement 3 is correct. PMRPY was announced on August 07, 2016 and is being implemented by the
Ministry of Labour and Employment through the Employees’ Provident Fund Organization (EPFO)
Subject) Social Schemes
Q.89)
Ans) b
Relevance (Why the question has been asked): Recently in the news, Utkarsh Bangla has got the
topmost award and emerged a winner in Capacity Building Category and Sabooj Sathi ranked in the
first five as a champion project in E- Government category in World Summit on the Information Society
(WSIS) prizes of the United Nations.
Source: https://timesofindia.indiatimes.com/city/kolkata/global-honour-for-sabooj-sathi-utkarsh-
bangla/articleshow/68087800.cms
https://wbsaboojsathi.gov.in/v2/about_scheme.php
https://timesofindia.indiatimes.com/india/there-is-growing-intolerance-in-society-says-
sc/articleshow/68836457.cms
Explanation) Option (b) is the correct answer.
● Sabooj Sathi. In 2015, Bengal launched the scheme that has since empowered lakhs of school
students (classes IX-XII) by providing them free bicycles. The objective of Sabooj Sathi, meaning
students’ companion, is to provide support to students, especially girls and thus reduce dropouts
in higher education.
● Utkarsh Bangla. It is a flagship scheme of West bengal governments to provide for placement
linked short-term skill trainings across the state of West Bengal and it also aims to provide
vocational training to school dropouts.
Subject) Social Schemes

Q.90)

ForumIAS Offline
2nd Floor, IAPL House, 19, Pusa Road, Karol Bagh, New Delhi – 110005| helpdesk@forumias.academy
 
PTS 2019 | Simulator Test 6 (Set A) - Solutions | ForumIAS
 
Ans) a
Source: https://indiankanoon.org/doc/1551554/
https://indiankanoon.org/doc/1256023/
Explanation) Option (a) – 1 and 2 only is the correct answer.
Statement 1 is correct: Under Article 39(d) Equal Pay for equal work for both men and women has been
envisaged
Statement 2 is correct: Under Article 43, “the State shall endeavour to promote cottage industries on
an individual or co-operative basis in rural areas”. Thus it has been envisaged in DPSP
Statement 3 is incorrect: Under Article 47, “the State shall endeavour to bring about prohibition of the
consumption except for medicinal purposes of intoxicating drinks and of drugs which are injurious to
health”. Thus complete prohibition has not been envisaged
Statement 4 is incorrect: It is envisaged in Fundamental duties Under Article 51A(f) of the constitution
and not under DPSP.
Subject) Polity

Q.91)
Ans) a
Source: https://natureconservation.pensoft.net/article/5625/
Explanation) According to the Indian Wildlife Protection Act, 1972, star tortoises fall inSchedule IV which
grants them the least level of protection. The species is deemed “vulnerable” by IUCN and falls in
Appendix II of the Convention against International Trade in Endangered Species of Flora and Fauna
(CITES).
Subject) Biodiversity

Q.92)
Ans) b
Explanation) Option (b) – 3 only is the correct answer.
Statement 1 is incorrect: Though Under Article 316 members of State Public Service Commission are
appointed by the Governor but Under Article 317 they can be removed only by President.
Statement 2 is incorrect: The Chairman or any other member of a Public Service Commission shall only
be removed from his office by order of the President on the ground of misbehaviour after the Supreme
Court, on reference being made to it by the President, has, on inquiry held in accordance with the
procedure prescribed in that behalf under Article 145, reported that the Chairman or such other member,
as the case may be, ought on any such ground to be removed.
CAG can be removed on like ground of Judge of Supreme Court, Thus removal process of members of
public service commission and CAG is different.
Statement 3 is correct: Under Article 320(4) of the Constitution, Public Service Commission need not be
consulted as respects the manner in which any provision referred to in clause ( 4 ) of Article 16 may be
made or as respects the manner in which effect may be given to the provisions of Article 335.
Subject) Polity

ForumIAS Offline
2nd Floor, IAPL House, 19, Pusa Road, Karol Bagh, New Delhi – 110005| helpdesk@forumias.academy
 
PTS 2019 | Simulator Test 6 (Set A) - Solutions | ForumIAS
 

Q.93)
Ans) d
Source: Spectrum: A Brief History of Modern India
Explanation) Option (d) – 3 – 1 – 2, is the correct answer.
Chittagong Armoury Raid - April 1930 (Surya Sen)
First Round Table Conference – November 1930-January 1931
Bhagat Singh, Rajguru, and Sukhdev executed on March 23, 1931.
Subject) Modern History

Q.94)
Ans) d
Source: Indian Economy by Shankar Ganesh Page# 147
Explanation) Option (d) – 1, 2 and 3 is the correct answer.
Statement 1 is correct: Increased disposable income means increase availability of money with the
individuals. Hence, the liquidity increases.
Statement 2 is correct: Cheap monetary policy means loan availability at very low interest rate at easy
terms. The borrowing tendency of public will increase thereby increasing liquidity in the market.
Statement 3 is correct: repayment of public debt borrowed by the government to public leaves people
with more money. Thereby increasing the liquidity.
Subject) Indian Economy

Q.95)
Ans) b
Source:https://m.timesofindia.com/india/gujarat-to-shift-lions-from-gir-to-
barda/amp_articleshow/66093394.cms
Explanation) Gujarat government has decided to translocate some the lions from the Gir forest to Barda
Dungar wildlife sanctuary. Barda Dungar is a sprawl of about 343 km and located 80 km from Gir in
Porbandar district. Barda along with Kuno Palpur in Madhya Pradesh and two other sanctuaries were
shortlisted as probable second homes for lions back in 1995. When Supreme Court directed the Gujarat
government to translocate the lions to Kuno Palpur in 2013, Gujrat worked at a feverish pitch to develop
a second habitat to host the lions within Gujarat.
Subject) Biodiversity

Q.96)
Ans) d
Explanation) Option (b) – 1 and 2 only is the correct answer.
Statement 1 is correct - Legislative Power of a state legislature is not suspended, it becomes subject to
the overriding power of the parliament. Thus, the proclamation does not suspend the state legislature, it
suspends the distribution of legislative powers between the union and the state.

ForumIAS Offline
2nd Floor, IAPL House, 19, Pusa Road, Karol Bagh, New Delhi – 110005| helpdesk@forumias.academy
 
PTS 2019 | Simulator Test 6 (Set A) - Solutions | ForumIAS
 
Statement 2 is correct - After the enactment of 44th Constitutional Amendment, Article 19 can be
suspended only when national emergency is declared on the ground of war or external aggression and
not on the ground of armed rebellion.
Statement 3 is incorrect - After the enactment of 44th Constitutional Amendment, President cannot
suspend the right to move the court for the enforcement of fundamental rights guaranteed by Articles 20
and 21
Subject) Polity

Q.97)
Ans) a
Explanation) Option (d) – 1 and 3 only is the correct answer.
Statement 1 is correct - Supreme Court in the Maenka Case in 1978 declared Prisoner’s right to have
the necessities of life as part of Article 21.
Statement 2 is incorrect - Right to life under Article 21 of the constitution of india does not include the
right to die. An assisted suicide and active euthanaisa are not legal.
Statement 3 is correct - The act of using ill equipped, barely literate youngsters in counter insurgency
activities is necessarily revelatory of disrespect for the life of the tribal youth and defiling of their human
dignity.
Subject) Polity

Q.98)
Ans) c
Source: 86, Khullar https://en.m.wikipedia.org/wiki/Great_Nicobar_Island
Explanation) Great Nicobar is the southernmost and largest of the Nicobar Islands of India. Little Nicobar
is only 34 km away. The island of Sumatra is located 180 km to the south of Great Nicobar.
Subject) Indian Geography

Q.99)
Ans) d
Exp)
Relevance (Why the question has been asked): Office of profit was in the news.
Source:
Explanation)
Subject) Polity

Q.100)
Ans) a
Exp)
Relevance (Why the question has been asked):
Source:

ForumIAS Offline
2nd Floor, IAPL House, 19, Pusa Road, Karol Bagh, New Delhi – 110005| helpdesk@forumias.academy
 
PTS 2019 | Simulator Test 6 (Set A) - Solutions | ForumIAS
 
Explanation)
Subject) Polity

ForumIAS Offline
2nd Floor, IAPL House, 19, Pusa Road, Karol Bagh, New Delhi – 110005| helpdesk@forumias.academy
 

You might also like